nzheng
Thanks Received: 0
Forum Guests
 
Posts: 10
Joined: May 18th, 2010
 
 
 

Q19 - There have been no new

by nzheng Tue May 18, 2010 4:44 pm

Could someone please explain why A is the correct answer? I eliminated this since in the argument it said that substituting IPV for most polio imunizations would cut the number of cases of vaccination-caused polio about in half, so it looked like a restatement of a premise.Thanks.
User avatar
 
ManhattanPrepLSAT1
Thanks Received: 1909
Atticus Finch
Atticus Finch
 
Posts: 2851
Joined: October 07th, 2009
 
This post thanked 3 times.
 
 

Re: Q19 - There have been no new

by ManhattanPrepLSAT1 Thu May 20, 2010 4:36 am

The conclusion of the argument here is that it is time to switch from OPV to IPV as the most commonly used polio vaccine.

The argument offers us the evidence that there would be fewer vaccination-caused cases of polio.

It's easy to read this piece of evidence and conclude that it makes sense to switch to the apparently safer IPV vaccine. The problem with this reasoning is that while IPV may cause fewer cases of polio, it may allow a significant number of naturally occurring cases of polio. So, overall, IPV could lead to there being an overall increase in the total number of cases of polio.

(A) weakens the argument by reminding us that the IPV vaccine could lead to increased naturally occurring cases of polio and so while there would be a reduction in vaccination-caused cases, there would still be an overall increase in the total number of cases of polio.
(B) is irrelevant. We care about the number of cases of polio caused by OPV, but not who gets those cases. Whether or not it's children doesn't really affect the argument.
(C) is irrelevant. The argument never claims the risk of contracting polio is high. It merely claims that the risk could be reduced by switching from OPV to IPV - which may or may not be true.
(D) is irrelevant. Preferences for one vaccine over another by certain countries does not give any measure of risk factors associated with each of the polio vaccines.
(E) is irrelevant. If IPV has this risk, similar to most vaccines, than IPV is no more dangerous than most vaccines. This does not provide a new threat that did not exist using OPV.
 
canylaw
Thanks Received: 0
Vinny Gambini
Vinny Gambini
 
Posts: 21
Joined: July 24th, 2011
 
 
 

Re: Q19 - : There have been no new cases..

by canylaw Sun Aug 21, 2011 2:28 pm

I understand that IPV will not address the prevailing problem to cure natural occurring polio (said implicitly) but how it will increase the number of natural occurring polio.

I thought A weakened the argument/core because there could be vaccine X best suited for the cause and the replacement for OPV.May with 100% polio free population.

:|
User avatar
 
ManhattanPrepLSAT1
Thanks Received: 1909
Atticus Finch
Atticus Finch
 
Posts: 2851
Joined: October 07th, 2009
 
This post thanked 1 time.
 
 

Re: Q19 - : There have been no new cases..

by ManhattanPrepLSAT1 Thu Aug 25, 2011 2:13 pm

canylaw Wrote:I understand that IPV will not address the prevailing problem to cure natural occurring polio (said implicitly) but how it will increase the number of natural occurring polio.

Good question. We don't really need to know how it will increase the number of cases of naturally occurring polio. The issue is that OPV causes 12 new cases of polio each year but prevents all naturally occurring polio. If we switch to IPV it may cut the number of cases of polio caused by the vaccine in half, but it may not be as potent, or may not cover as many strains of polio, who know what? It doesn't matter. If there are naturally occurring cases of polio (say more than 6 per year), than switching to IPV would result in a net gain of new cases of polio every year. This would give us reason to question whether it's a good idea to switch to IPV.

Does that answer your question?
canylaw Wrote:I thought A weakened the argument/core because there could be vaccine X best suited for the cause and the replacement for OPV.May with 100% polio free population.

I don't think we need to consider alternative vaccines. That is an important thought though on many Weaken questions, so don't think that providing some alternative will always be wrong. That's just not suggested by answer choice (A).

Hope that helps!
 
timmydoeslsat
Thanks Received: 887
Atticus Finch
Atticus Finch
 
Posts: 1136
Joined: June 20th, 2011
 
This post thanked 5 times.
 
trophy
Most Thanked
trophy
First Responder
 

Re: Q19 - : There have been no new cases..

by timmydoeslsat Thu Aug 25, 2011 4:40 pm

I just looked at this question and the only answer I think is tempting is B.

This is because it makes the test taker want to say, "Oh, maybe this would impact the IPV just like the OPV. It really isn't the vaccine, but the susceptibility of certain children!"

However, this can be refuted with a couple of points.

1) Choice B states that OPV still caused those cases.

2) The stimulus tells us that IPV will cut the cases of vaccine-induced polio cases in half. So really this statement absorbs this concern.

Like Matt said, we simply do not know enough about IPV to warrant concluding a switch. And (A) shows this by suggesting that IPV would allow natural polio to become a concern again, whereas it was not with OPV.
 
Raiderblue17
Thanks Received: 4
Forum Guests
 
Posts: 26
Joined: August 10th, 2011
 
 
 

Re: Q19 - : There have been no new cases..

by Raiderblue17 Sun Sep 18, 2011 3:07 pm

Id like to throw in my two cents to why B, in my mind, is actually the correct answer.

So OPV should be replaced b/c it caused approx. 12 new cases of polio. We also know that IPV would cut it in half (6 cases)

Now "B" works because it says that the vast majority (over 6 is a majority) of OPV's polio causing was not the fault of the immunization, but the fact that children were weak in immunity.

So OPV really caused let's just say 5 polio problems
IPV causes let's say 6 (half) So really OPV is the better choice because IPV causes just as much polio.

Thoughts
 
zainrizvi
Thanks Received: 16
Atticus Finch
Atticus Finch
 
Posts: 171
Joined: July 19th, 2011
 
 
trophy
First Responder
 

Re: PT 47 S1 Q19: There have been no new cases..

by zainrizvi Tue Oct 18, 2011 7:56 pm

mshermn Wrote:The conclusion of the argument here is that it is time to switch from OPV to IPV as the most commonly used polio vaccine.

The argument offers us the evidence that there would be fewer vaccination-caused cases of polio.

It's easy to read this piece of evidence and conclude that it makes sense to switch to the apparently safer IPV vaccine. The problem with this reasoning is that while IPV may cause fewer cases of polio, it may allow a significant number of naturally occurring cases of polio. So, overall, IPV could lead to there being an overall increase in the total number of cases of polio.

(A) weakens the argument by reminding us that the IPV vaccine could lead to increased naturally occurring cases of polio and so while there would be a reduction in vaccination-caused cases, there would still be an overall increase in the total number of cases of polio.
(B) is irrelevant. We care about the number of cases of polio caused by OPV, but not who gets those cases. Whether or not it's children doesn't really affect the argument.
(C) is irrelevant. The argument never claims the risk of contracting polio is high. It merely claims that the risk could be reduced by switching from OPV to IPV - which may or may not be true.
(D) is irrelevant. Preferences for one vaccine over another by certain countries does not give any measure of risk factors associated with each of the polio vaccines.
(E) is irrelevant. If IPV has this risk, similar to most vaccines, than IPV is no more dangerous than most vaccines. This does not provide a new threat that did not exist using OPV.


How can we make the assumption that OPV follows under the "most vaccines" category?

I've had the same problem with Q6 of this section, which was a paradox question; we had to assume that this particular model was included in the computer models that "often fail".

Is this because of the nature of question(weaken) allows us for a bit of vague language? If so, can you specify in which questions you are expected to be very precise in language? And which ones you have leeway?

I would think that must be true and necessary assumption questions would require the most support. But I'm not sure about the others.
 
zainrizvi
Thanks Received: 16
Atticus Finch
Atticus Finch
 
Posts: 171
Joined: July 19th, 2011
 
 
trophy
First Responder
 

Re: Q19 - : There have been no new cases..

by zainrizvi Sun Nov 20, 2011 11:07 am

What if the answer choice had said IPV, like SOME vaccines, carries a slight risk of other side effects... would that be weakening the argument?

Some could encompass all so I don't think it would.

But what if it it said LESS than half vaccines? I think then it might be more of a weakener as it suggests that there is less chance for OPV to to also have that problem.


(I realized these are all hypotheticals so the answers are in a huge shade of gray, but I would appreciate in anyone could confirm these thoughts)
 
goriano
Thanks Received: 12
Atticus Finch
Atticus Finch
 
Posts: 113
Joined: December 03rd, 2011
 
 
 

Re: Q19 - : There have been no new cases..

by goriano Sat Jan 07, 2012 2:52 pm

Raiderblue17 Wrote:Id like to throw in my two cents to why B, in my mind, is actually the correct answer.


Your reasoning is faulty because you've confused causation with correlation. Let me explain this.

Raiderblue17 Wrote: The vast majority (over 6 is a majority) of OPV's polio causing was not the fault of the immunization, but the fact that children were weak in immunity.


You're essentially saying that a weakened immunity CAUSED those cases of polio. But that is not what (B) is saying! (B) is saying that while OPV CAUSED the vast majority of polio cases, they were CORRELATED with children having preexisting immunodeficiency disorders.
 
jamiejames
Thanks Received: 3
Atticus Finch
Atticus Finch
 
Posts: 116
Joined: September 17th, 2011
 
 
 

Re: Q19 - : There have been no new cases..

by jamiejames Tue Apr 10, 2012 6:00 pm

I chose A as the answer, but I didn't want to.

My problem is this:

The passage accepts that while changing over, the IPV may still cause polio in some people, by saying there would be fewer cases. I thought that A was wrong because it was simply restating part of the passage, but in a different way, and I didn't think we were allowed to use premises as answers to strengthen/weaken questions.
 
goriano
Thanks Received: 12
Atticus Finch
Atticus Finch
 
Posts: 113
Joined: December 03rd, 2011
 
This post thanked 1 time.
 
 

Re: Q19 - There have been no new cases

by goriano Wed Apr 11, 2012 2:15 pm

jeastman Wrote:I chose A as the answer, but I didn't want to.

My problem is this:

The passage accepts that while changing over, the IPV may still cause polio in some people, by saying there would be fewer cases. I thought that A was wrong because it was simply restating part of the passage, but in a different way, and I didn't think we were allowed to use premises as answers to strengthen/weaken questions.


I think you're misquoting what (A) is saying. The new cases of polio mentioned in (A) are NATURALLY-caused polio, which is different from VACCINATION-caused polio.
User avatar
 
ManhattanPrepLSAT1
Thanks Received: 1909
Atticus Finch
Atticus Finch
 
Posts: 2851
Joined: October 07th, 2009
 
 
 

Re: Q19 - There have been no new cases

by ManhattanPrepLSAT1 Fri Apr 13, 2012 7:56 pm

Good point goriano! There is a difference between the naturally occurring polio discussed in answer choice (A) and the vaccine-caused polio discussed in the stimulus.

Answer choice (A) gets to the issue. What if the new vaccine is weaker or less effective? Then even if it causes less cases of polio itself, it may still lead to an increase in the overall number of polio cases that occur.
 
slimjimsquinn
Thanks Received: 1
Forum Guests
 
Posts: 43
Joined: February 11th, 2012
 
 
 

Re: Q19 - There have been no new

by slimjimsquinn Sat Sep 15, 2012 4:01 pm

Stimulus:
1. no new cases of naturally occurring polio
2. approximately 12 new cases of occurring polio each year.

Isn't it a bit of a leap to make a causal assumption that IPV prevents polio? Or is that the flaw?

Why couldn't we assume IPV also prevents polio? If the assumption is between all vaccines and naturally occurring, wouldn't that put IPV and OPV on the same footing?
 
ptewarie
Thanks Received: 36
Forum Guests
 
Posts: 38
Joined: October 01st, 2012
 
 
 

Re: Q19 - There have been no new

by ptewarie Mon Oct 01, 2012 2:06 pm

A few key things to remember will help you understand this question and why B is false, although it seems attractive.

1. B notes that OPV occurred in children with pre-existing immune disorders.
Does this mean that OPV is absolutely & definitely not the causing the polio for these children? There is no way we can know for sure that DESPITE the immune disorder, the OPV is not
a) Fully causing it
b) Compounding the issue

Even if we assume that the immune disorder makes polio contraction more likely, we can not infer that OPV does NOT make it any worse.

2. Look closely at the stimulus. The recommendation is the last line states " its time to switch away from OPV as the most commonly used polio vaccine for North American CHILDREN.

Notice they are referring to ALL CHILDREN in North American, which can possibly include those children with immune disorders.

If we believe B to be true, and that that those kids with pre-existing immune disorders are likely to get OPV, than it would be a good idea to avoid giving them the OPV shot
This would actually strengthen the author's case.

Bottom line: this problem is difficult, but always look CLOSELY the scope of the stimulus and actively read every single word it says. Our brains are wired to skip, and combine certain words. This does not work in the LSAT.
Here, a close reader would have suspected there is something wrong when the author goes from:
There have been 0 new cases of naturally occurring Polio to saying there are 12 new cases of polio each year. A bulb must have lit been like: What??? Then you'd realize there are 2 categories the author is referring to

1. Natural occuring Polio
2. Vaccine caused Polio

If HPV decreases vaccine caused Polio but raises the rate of natural occurring Polio, a case can be made against its use. As of now, with OPV use, we see 0 cases of natural occurring Polio.

ie:

OPV: 0 Natural occurring VS 12 Vaccine Occuring
HPV: 7(random) Natural occuring vs 6 Vaccine Occurring

This answer still has holes( if we assume only 1 natural occuring and 6 vaccine occuring, HPV would be safer), but remember, it is a weaken question so you don't have to destroy the argument, only show that it's foundations are weak and it can be open to attack

Hope that helps
 
WesleyC316
Thanks Received: 3
Jackie Chiles
Jackie Chiles
 
Posts: 40
Joined: March 19th, 2018
Location: Shanghai
 
 
 

Re: Q19 - There have been no new

by WesleyC316 Sun Apr 15, 2018 10:03 am

One thing I don't understand about A is the phrase "a few". What if the number of new cases is just 1 or 2, which is clearly a lot fewer than 6, then we still should choose IPV, shouldn't we? Or is it because the argument says "clearly", which is a strong tone implicating that IPV is better than OPV in every aspect?
In that case A does weaken the argument, because it shows there's at least one aspect in which OPV outweighs IPV. Am I getting this right? Someone please break it down for me. Many thanks!

#Edit After studying for another 2 months, I've realized that's how things work in LSAT. Weakening & Strengthening questions are the hardest because the right answer choices can be subtle and seemingly out of scope. For this question, pointing out one of the weaknesses is enough.
 
Nathanael M864
Thanks Received: 0
Vinny Gambini
Vinny Gambini
 
Posts: 1
Joined: March 23rd, 2021
 
 
 

Re: Q19 - There have been no new

by Nathanael M864 Tue Mar 23, 2021 2:12 am

I chose B because I didn't want to leave a blank, but in saying that I really didn't like any of the options at all. For A, we are told OPV causes 12 cases + no natural cases of Polio each year while IPV causes only 6 + a "few" natural cases.

In this context, it appears to me that a "few" could refer to any single digit number given the values provided of the cases in the stimulus. So if we take A to be true, then IPV total cases to be 6 + (2) through 6 + (5), could reasonably be a strengthener in my view. So could this choice not be a strengthen, neutral ((6) + (6)), or weaken, thus invalidating it?

Are we simply meant to infer that a "few" is defined in relation to the total population of North America, presumably some large number in the hundreds, if not thousands, and NOT in relation to the value of the "artificial" cases, 12 and 6 by the OPV and IPV respectively?

Cheers.